S&Jv8,p27.01: Add missing period to the question
[course.git] / latex / problems / Serway_and_Jewett_8 / problem27.01.tex
index 2e738ba4e8d86d2b1c7f333345047de19b48aa10..5c1e1940e995a54761d4ed20cdf462eed4c6a933 100644 (file)
@@ -1,5 +1,5 @@
 \begin{problem*}{27.1}
-A proton beam in an accelerator carries a current of $125\U{$\mu$A}$
+A proton beam in an accelerator carries a current of $125\U{$\mu$A}$.
 If the beam is incident on a target, how many protons strike the
 target in a period of $23.0\U{s}$.
 \end{problem*}